Difference between revisions of "2010 AMC 10A Problems/Problem 14"

m (Problem)
(Redirected page to 2010 AMC 12A Problems/Problem 8)
(Tag: New redirect)
 
(6 intermediate revisions by 2 users not shown)
Line 1: Line 1:
== Problem ==
+
#REDIRECT [[2010_AMC_12A_Problems/Problem_8]]
Triangle <math>ABC</math> has <math>AB=2 \cdot AC</math>. Let <math>D</math> and <math>E</math> be on <math>\overline{AB}</math> and <math>\overline{BC}</math>, respectively, such that <math>\angle BAE = \angle ACD</math>. Let <math>F</math> be the intersection of segments <math>AE</math> and <math>CD</math>, and suppose that <math>\triangle CFE</math> is equilateral. What is <math>\angle ACB</math>?
 
 
 
<math>\textbf{(A)}\ 60^\circ \qquad \textbf{(B)}\ 75^\circ \qquad \textbf{(C)}\ 90^\circ \qquad \textbf{(D)}\ 105^\circ \qquad \textbf{(E)}\ 120^\circ</math>
 
 
 
== Solution ==
 
Let <math>\angle BAE = \angle ACD = x</math>.
 
 
 
<cmath>\begin{align*}\angle BCD &= \angle AEC = 60^\circ\\
 
\angle EAC + \angle FCA + \angle ECF + \angle AEC &= \angle EAC + x + 60^\circ + 60^\circ = 180^\circ\\
 
\angle EAC &= 60^\circ - x\\
 
\angle BAC &= \angle EAC + \angle BAE = 60^\circ - x + x = 60^\circ\end{align*}</cmath>
 
 
 
Since <math>\frac{AC}{AB} = \frac{1}{2}</math>, <math>\angle BCA = \boxed{90^\circ\ \textbf{(C)}}</math>
 
 
 
== See also ==
 
{{AMC10 box|year=2010|num-b=13|num-a=15|ab=A}}
 
 
 
[[Category:Introductory Geometry Problems]]
 
{{MAA Notice}}
 

Latest revision as of 13:24, 26 May 2020